What is the electric field on (or near) the rim of a uniformly charged disk? I know how to find the electric field along the central axis at a certain point away on that axis from the uniformly charged disk (negligible thickness), but there was an example in my E&M textbook (Purcell and Morin) where they calculated the electric potential on the rim of said disk due to other charges on the disk and it ended up being $$\phi = \frac{\sigma a}{\pi \epsilon_{0}}. $$ By using the Gradient theorem saying that electric field is the negative gradient of electric potential, is it true that it is just zero as you take the partial of the above electric potential with respect to $r$? How can this be if the electric field points outwards/"up" from the center of the disk towards the rim pushing charges upwards towards the edges of the disk -- isn't this nonzero, or do all the vectors cancel out to 0 in which case how are those charges pushed to the rim in the first place if the net electric field at that point is 0?
-
1If you know the potential at only one location — on the rim — you can’t conclude anything about the field. You would have to know the potential in a region near the rim so that you can take the gradient of that function of $r$ and $z$. – G. Smith Mar 18 '20 at 17:28
-
Hmm, so is there any way to find the electric field another way at the rim of the disk? – Parzivalz13 Mar 18 '20 at 17:30
-
Both the potential and the field at any point for any charge distribution can be found by integrating. The integral for the potential is usually easier. For the disk I believe both are hard, but I think there is an analytic solution. I’ll see if I can find it. – G. Smith Mar 18 '20 at 17:35
-
I found this. It doesn’t manage to get an analytic solution but it reduces the integrals to (3) or (11). I have a feeling, though, that this problem is treated in Jackson. – G. Smith Mar 18 '20 at 17:43
-
If you only care about the field in the plane of the disk, you can take the gradient of (13). – G. Smith Mar 18 '20 at 17:46
-
(12) also looks promising for taking the gradient. The resulting integral might simplify to give an expression for the field at the rim. – G. Smith Mar 18 '20 at 17:54
-
Thank you! I don't really understand that math (Bessel functions, etc.) yet -- just a freshman undergrad physics student, but I'll spend some time looking at it later – Parzivalz13 Mar 18 '20 at 18:34
-
After looking at the behavior of the potential in the plane, I believe that the field at the rim is in fact infinite. The potential in the plane is well-defined and expressible in terms of elliptic functions but it seems to have an infinite slope at $\rho=a$. This is probably why I can't find any calculations of the field there. – G. Smith Mar 18 '20 at 20:14
-
It is finite https://physics.stackexchange.com/questions/528743/why-is-the-electric-field-at-the-edge-of-a-uniformly-charged-disk-infinite/528802#528802 – Mar 18 '20 at 21:09
-
@Wolphramjonny No, sorry, it isn’t. – G. Smith Mar 18 '20 at 21:52
1 Answers
The field at the rim is infinite.
The simplest way to calculate the potential and the field at the rim is to introduce Cartesian coordinates in which the center of the disk is not at the origin but at $(a,0)$. Then we will calculate quantities at the origin, which is a typical point on the rim.
To actually do the integrals, use polar coordinates $(r,\theta)$. To integrate over the disk, the $\theta$ coordinate will range from $-\pi/2$ to $\pi/2$, and the $r$ coordinate can be shown to range from $0$ to $2a\cos\theta$. (To see this, draw a diagram and do the trig.)
The potential at the origin (on the rim) is
$$\varphi=k_e\int\frac{\sigma\,dA}{r}$$
where $k_e$ is the electric constant ($1/4\pi\epsilon_0$ in SI units; $1$ in Gaussian units). The area element is $dA=r\,dr\,d\theta.$ So we get the known potential
$$\varphi=k_e\sigma\int_{-\pi/2}^{\pi/2}d\theta\int_0^{2a\cos\theta}dr=4k_e\sigma a.$$
The field integral at the origin (on the rim) is
$$E_x=-k_e\int\frac{\sigma\,dA \cos\theta}{r^2}$$
which is divergent:
$$E_x=-k_e\sigma\int_{-\pi/2}^{\pi/2}\cos\theta\,d\theta\int_0^{2a\cos\theta}\frac{dr}{r}=+\infty.$$
ADDENDUM:
For a more complicated but more complete approach...
In cylindrical coordinates $(\rho,\phi,z)$ centered on the disk, the potential at any point can be shown to be
$$\varphi(\rho,z)=2\pi k_e\sigma a\int_0^\infty\frac{dk}{k}e^{-k|z|}J_0(k\rho)J_1(ka)$$
where the $J_m(x)$ are Bessel functions of the first kind.
Special cases are easy to compute:
The potential at the center of the disk is
$$\varphi(0,0)=2\pi k_e\sigma a\int_0^\infty\frac{dk}{k}J_1(ka)=2\pi k_e\sigma a\,\big(1\big).$$
The potential at the rim is lower by a factor of $2/\pi$:
$$\varphi(a,0)=2\pi k_e\sigma a\int_0^\infty\frac{dk}{k}J_0(ka)J_1(ka)=2\pi k_e\sigma a\,\left(\frac{2}{\pi}\right).$$
The potential along the $z$-axis (a standard homework calculation by a simpler approach) is
$$\varphi(0,z)=2\pi k_e\sigma a\int_0^\infty\frac{dk}{k}e^{-k|z|}J_1(ka)=2\pi k_e\sigma a\,\left(\frac{\sqrt{a^2+z^2}-|z|}{a}\right).$$
In the disk’s plane where $z=0$, the integral gives a function of $\rho$,
$$\varphi(\rho,0)=2\pi k_e\sigma a\left\{ \begin{array}{ll} \frac{2}{\pi}E\left(\frac{\rho^2}{a^2}\right), & \rho<a \\ \frac{2}{\pi}\left[\frac{\rho}{a}E(\frac{a^2}{\rho^2})+\frac{a^2-\rho^2}{a\rho}K(\frac{a^2}{\rho^2})\right], & \rho>a, \end{array} \right.$$
where $K(k)$ and $E(k)$ are complete elliptic integrals of the first and second kind.
This can be shown to have infinite slope at $\rho=a$. However, this is not particularly obvious when graphing the potential, which looks like this, where the horizontal axis is $\rho/a$ and the vertical axis is in units of $2\pi k_e\sigma a$:
One can also see that the field is infinite by computing the gradient of the potential inside the integral. That procedure produces for $\rho=a$ and $z=0$ an integral involving Bessel functions which is divergent.
ADDENDUM 2:
For even more completeness...
One finds that the field in the $z=0$ plane is given by
$$E_\rho=2\pi k_e\sigma a\left\{ \begin{array}{ll} \frac{2}{\pi\rho}\left[K(\frac{\rho^2}{a^2})-E(\frac{\rho^2}{a^2})\right], & \rho<a \\ \frac{2}{\pi a}\left[K(\frac{a^2}{\rho^2})-E(\frac{a^2}{\rho^2})\right], & \rho>a \end{array} \right.$$
The function to the right of the brace looks like this, where the horizontal axis is $\rho/a$:
The divergence at the rim is because $K(1)$ is infinite.

- 51,534
-
https://physics.stackexchange.com/questions/528743/why-is-the-electric-field-at-the-edge-of-a-uniformly-charged-disk-infinite/528802#528802 – Mar 18 '20 at 21:07
-
-
-
-
thanks to all, I will review the answers and see what is wrong in the one I linked – Mar 18 '20 at 22:00
-
Down vote most likely comes from someone being upset that you answered a homework-like question with a solution-like answer. – BioPhysicist Mar 19 '20 at 14:25
-
It is not a homework question. It is a curiosity of mine as I was reading my textbook – Parzivalz13 Mar 21 '20 at 03:04
-
Apologies for tagging it as a HW question, I only did so because I thought it would get a faster response time which is wrong on my part. Sorry about that, it won't happen again. – Parzivalz13 Mar 21 '20 at 03:06
-
Having both taken and taught multiple EM courses, I can say that in my experience this is not a common homework question. But of course the two people who have voted to close it as homework-like are entitled to have a different opinion. I think it is a very interesting question with a surprising answer. – G. Smith Mar 21 '20 at 03:10
-
I also note that of three other PSE questions on this topic (see Wolfram Jonny’s link, and the two links in that), not one received a single vote to close as homework-like, and not one got a definitive answer until now. – G. Smith Mar 21 '20 at 03:17
-
Hmm @G. smith, I have a doubt on this step:$$\varphi=k_e\sigma\int_{-\pi/2}^{\pi/2}d\theta\int_0^{2a\cos\theta}dr=4k_e\sigma a.$$ If the bounds of r are dependent on theta , how did you factor the integral? I think so you would have to integrate with r , then get a function with theta which you then integrate with respect to theta – tryst with freedom May 22 '21 at 17:15
-